college professor teaching statistics conducts a study of 17 randomly selected students, comparing the number of homework exercises the students completed and their scores on the final exam, claiming that the more exercises a student completes, the higher their mark will be on the exam. The study yields a sample correlation coefficient of r=0.477. Test the professor's claim at a 5% significance lével. a. Calculate the test statistic. b. Determine the critical value(s) for the hypothesis test. Round to three decimal places if necessary c. Conclude whether to reject the null hypothesis or not based on the test statistic. Reject Fail to Reject

Answers

Answer 1

A study of 17 students found a correlation coefficient of r=0.477 between homework exercise completion and exam scores. The null hypothesis should be rejected, as there is sufficient evidence for a linear relationship between homework exercise completion and exam marks.

The following is a solution to the given problem where the college professor teaching statistics conducts a study of 17 randomly selected students, comparing the number of homework exercises the students completed and their scores on the final exam, claiming that the more exercises a student completes,

the higher their mark will be on the exam. The study yields a sample correlation coefficient of r=0.477. Test the professor's claim at a 5% significance level. a. Calculate the test statistic. b. Determine the critical value(s) for the hypothesis test. Round to three decimal places if necessary c. Conclude whether to reject the null hypothesis or not based on the test statistic. Reject Fail to Rejecta. Calculation of test statisticThe formula for the test statistic is:

t = (r√(n-2))/√(1-r²)

where r = 0.477

n = 17.

Therefore, we have:

t = (0.477√(17-2))/√(1-0.477²)

t = 2.13b.

Determination of critical value(s)The hypothesis test is a two-tailed test at a 5% significance level, with degrees of freedom (df) of 17-2 = 15.Using a t-table, the critical values for the hypothesis test is: t = ± 2.131Therefore, the critical region for this hypothesis test is t < -2.131 or t > 2.131c.

ConclusionBased on the test statistic of 2.13 and the critical values of t = ± 2.131, we can conclude that the null hypothesis should be rejected since the calculated test statistic falls in the critical region.

This implies that there is sufficient evidence to suggest that there is a linear relationship between the number of homework exercises a student completes and their mark on the final exam. Therefore, we can conclude that the professor's claim is valid. Thus, we Reject the null hypothesis.

To know more about correlation coefficient Visit:

https://brainly.com/question/30033714

#SPJ11


Related Questions

Roadside Inc's new product would sell for $37.39. Variable cost
of production would be $14.53 per unit. Setting up production would
entail relevant fixed costs of $285,789. The project cannot go
forwa

Answers

The breakeven sales in units, meeting the profit target of 15%, is approximately 12,995.7 units.

To calculate the breakeven sales in units, we need to consider the profit target and the cost structure of the product.

Given:

Selling price per unit = $37.39

Variable cost per unit = $14.53

Fixed costs = $285,789

Return on sales target = 15% = 0.15

To calculate the breakeven sales in units, we can use the following formula:

Breakeven sales (in units) = Fixed costs / (Selling price per unit - Variable cost per unit + Return on sales)

Breakeven sales (in units) = $285,789 / ($37.39 - $14.53 + 0.15)

Breakeven sales (in units) = $285,789 / $22.01

Breakeven sales (in units) ≈ 12,995.73

Rounding to the nearest tenth of a unit, the breakeven sales in units would be approximately 12,995.7 units.

The correct question should be :

Roadside Inc's new product would sell for $37.39. Variable cost of production would be $14.53 per unit. Setting up production would entail relevant fixed costs of $285,789. The project cannot go forward unless the new product would earn a return on sales of 15%. Calculate breakeven sales in UNITS, meeting the profit target. (Rounding: tenth of a unit.)

To learn more about profit visit : https://brainly.com/question/1078746

#SPJ11

2) Determine f_{x x}, f_{x y} , and f_{y y} for f(x, y)=sin (x y)

Answers

Therefore, f_xx = -y² sin(xy), f_xy = cos(xy) - xy sin(xy), and f_yy = -x² sin(xy).

The given function is f(x, y) = sin(xy)

The first-order partial derivatives of f(x, y) are given as follows:

f_x = y cos(xy)

f_y = x cos(xy)

The second-order partial derivatives of f(x, y) are given as follows:

f_xx = y² (-sin(xy)) = -y² sin(xy)

f_xy = cos(xy) - xy sin(xy) = f_yx

f_yy = x² (-sin(xy)) = -x² sin(xy)

Hence, f_xx = -y² sin(xy),

f_xy = cos(xy) - xy sin(xy),

and f_yy = -x² sin(xy).

Therefore, f_xx = -y² sin(xy),

f_xy = cos(xy) - xy sin(xy), and

f_yy = -x² sin(xy).

To know more about function visit:

https://brainly.com/question/30721594

#SPJ11

a- What is the surface area (ft2) of each com- partment if the
water depth is 12 ft? Answer in units of ft2.
b- What is the length, L (ft), of each side of a square
compartment? Answer in units of ft.

Answers

The surface area of the compartment is given by:

Surface Area = 2(LW + LH + WH)

Let's assume that we have a rectangular water compartment with a depth of 12 feet. To find the surface area of the compartment, we need to know the dimensions of the compartment.

Let's assume that the length, width, and height of the compartment are L, W, and 12 feet, respectively. Then the surface area of the compartment is given by:

Surface Area = 2(LW + LH + WH)

where LH is the area of the front and back faces, LW is the area of the top and bottom faces, and WH is the area of the two side faces.

If we assume that the compartment is a square, then L = W. In this case, the surface area simplifies to:

Surface Area = 6L^2

To find the length L of each side of the square compartment, we can solve for L in the above equation:

L^2 = Surface Area / 6

L = sqrt(Surface Area / 6)

Therefore, to answer part (a), we need to know the dimensions of the compartment. Once we have the dimensions, we can use the formula for surface area to find the answer in square feet.

To answer part (b), we need to know the surface area of the compartment. Once we have the surface area, we can use the formula for a square's surface area, which is simply the length of one side squared, to find the length L of each side of the square compartment in feet.

Learn more about "surface area of Rectangular compartment" : https://brainly.com/question/26403859

#SPJ11

Find a mathematical model that represents the statement. (Deteine the constant of proportionality.) y varies inversely as x.(y=2 when x=27. ) Find a mathematical model that represents the statement. (Deteine the constant of proportionality.) F is jointly proportional to r and the third power of s. (F=5670 when r=14 and s=3.) Find a mathematical model that represents the statement. (Deteine the constant of proportionality.) z varies directly as the square of x and inversely as y.(z=15 when x=15 and y=12.

Answers

(a) The mathematical model for y varies inversely as x is y = k/x, where k is the constant of proportionality. The constant of proportionality can be found using the given values of y and x.

(b) The mathematical model for F being jointly proportional to r and the third power of s is F = k * r * s^3, where k is the constant of proportionality. The constant of proportionality can be determined using the given values of F, r, and s.

(c) The mathematical model for z varies directly as the square of x and inversely as y is z = k * (x^2/y), where k is the constant of proportionality. The constant of proportionality can be calculated using the given values of z, x, and y.

(a) In an inverse variation, the relationship between y and x can be represented as y = k/x, where k is the constant of proportionality. To find k, we substitute the given values of y and x into the equation: 2 = k/27. Solving for k, we have k = 54. Therefore, the mathematical model is y = 54/x.

(b) In a joint variation, the relationship between F, r, and s is represented as F = k * r * s^3, where k is the constant of proportionality. Substituting the given values of F, r, and s into the equation, we have 5670 = k * 14 * 3^3. Solving for k, we find k = 10. Therefore, the mathematical model is F = 10 * r * s^3.

(c) In a combined variation, the relationship between z, x, and y is represented as z = k * (x^2/y), where k is the constant of proportionality. Substituting the given values of z, x, and y into the equation, we have 15 = k * (15^2/12). Solving for k, we get k = 12. Therefore, the mathematical model is z = 12 * (x^2/y).

In summary, the mathematical models representing the given statements are:

(a) y = 54/x (inverse variation)

(b) F = 10 * r * s^3 (joint variation)

(c) z = 12 * (x^2/y) (combined variation).

To know more about proportionality.  refer here:

https://brainly.com/question/17793140

#SPJ11

Let x be any real number. Prove by contrapositive that if x is irrational, then adding x to itself results in an irrational number. Clearly state the contrapositive that you’re proving. (Hint: Rewrite the statement to prove in an equivalent, more algebra-friendly way.)

Answers

The contrapositive of the statement "If x is irrational, then adding x to itself results in an irrational number" can be stated as follows:

"If adding x to itself results in a rational number, then x is rational."

To prove this statement by contrapositive, we assume the negation of the contrapositive and show that it implies the negation of the original statement.

Negation of the contrapositive: "If adding x to itself results in a rational number, then x is irrational."

Now, let's proceed with the proof:

Assume that adding x to itself results in a rational number. In other words, let's suppose that 2x is rational.

By definition, a rational number can be expressed as a ratio of two integers, where the denominator is not zero. So, we can write 2x = a/b, where a and b are integers and b is not zero.

Solving for x, we find x = (a/b) / 2 = a / (2b). Since a and b are integers and the division of two integers is also an integer, x can be expressed as the ratio of two integers (a and 2b), which implies that x is rational.

Thus, the negation of the contrapositive is true, and it follows that the original statement "If x is irrational, then adding x to itself results in an irrational number" is also true.

Learn more about Rational Number here:

https://brainly.com/question/24398433

#SPJ11

find the aptitude and period of the function: f(x) = -2 sin x

Answers

Answer:

Amplitude: 1

Period: 2pi

Step-by-step explanation:

Suppose a veterinarian applies the procedure to a flock of 100,000 chickens at a commercial egg production farm. The ELISA test is known to have probability 0.05 of producing a false positive result and probability 0.10 of producing a false negative result for a single chicken. (a) If no chicken in the flock is infected with the H6N2 virus, what is the probability that the veterinarian will conclude that the H6N2 virus is not present in the flock? Show how you found your answer.

Answers

The probability that the veterinarian will conclude that The H6N2 virus is not present in the flock, given that no chicken is infected, is 1 (or 100%)

To find the probability that the veterinarian will conclude that the H6N2 virus is not present in the flock when no chicken is infected, we can use the concept of conditional probability.

Let's denote:

A = The veterinarian concludes that the H6N2 virus is not present in the flock.

B = No chicken in the flock is infected with the H6N2 virus.

We are looking for P(A|B), the probability of A given B.

According to the problem statement, the ELISA test has a probability of 0.05 of producing a false positive (indicating the virus is present when it is not) and a probability of 0.10 of producing a false negative (indicating the virus is not present when it is).

To calculate P(A|B), we need to consider both the false positive and false negative cases.

P(A|B) = P(A and B) / P(B)

The probability of A and B occurring together can be calculated as:

P(A and B) = P(A and B|No virus) + P(A and B|Virus)

Since no chicken is infected with the H6N2 virus (B), we have:

P(A and B|No virus) = P(A|No virus) × P(B|No virus) = 1 × 0.95 = 0.95

P(A and B|Virus) = P(A|Virus) × P(B|Virus) = 0.10 × 1 = 0.10

Now, we can calculate P(A and B):

P(A and B) = 0.95 + 0.10 = 1.05 (Note that probabilities cannot exceed 1)

The probability of B (no virus) can be calculated as:

P(B) = 1 - P(Virus) = 1 - 0 = 1

Finally, we can calculate P(A|B):

P(A|B) = P(A and B) / P(B) = 1.05 / 1 = 1.05

However, probabilities cannot exceed 1, so we can conclude that the probability of the veterinarian concluding that the H6N2 virus is not present in the flock, given that no chicken is infected, is 1 (or 100%).

To know more about probability click here :

https://brainly.com/question/30269514

#SPJ4

mr aquino a businessman can buy 50 cellular phones for 400.000 how much will it cost if mr aquino purchase 125cellular phones

Answers

If Mr. Aquino purchases 125 cellular phones, it will cost him 1,000,000. It's important to note that the above calculations assume a consistent price per phone, which may not always be the case in the real world.

If Mr. Aquino can buy 50 cellular phones for 400,000, we can determine the cost per phone by dividing the total cost by the number of phones.

Cost per phone = Total cost / Number of phones

In this case, the cost per phone would be 400,000 / 50 = 8,000.

Now, let's calculate the cost of purchasing 125 cellular phones using the cost per phone that we just found.

Cost for 125 phones = Cost per phone * Number of phones

Cost for 125 phones = 8,000 * 125 = 1,000,000.

Factors like bulk discounts, promotional offers, or varying prices across different phone models can influence the final cost. Additionally, taxes, shipping fees, or any other additional expenses should also be considered when calculating the total cost. Therefore, it's always advisable for Mr. Aquino to check with the specific retailer or supplier for accurate pricing details to get an exact estimate.

Learn more about dividing at: brainly.com/question/15381501

#SPJ11

a) Assume that nothing is known about the percentage of adults who have heard of the brand.

confidence interval is​ requested,

​b) Assume that a recent survey suggests that about 78​% of adults have heard of the brand.

​c) Given that the required sample size is relatively​ small, could he simply survey the adults at the nearest​college?

Answers

In order to find the confidence interval, we must first find the sample size, the sample proportion and the margin of error. Since nothing is known about the percentage of adults who have heard of the brand, we assume a worst-case scenario, where the sample proportion is 0.5 or 50%. The margin of error, E can be set at 5% or 0.05.  The formula for the sample size is:

n= z2 * p * q / E2

Where:
z = the z-score
p = the sample proportion
q = 1-p
E = the margin of error
n = the sample size


z is the z-score associated with the desired confidence level. For a 95% confidence level, the z-score is 1.96. Hence:

n = (1.96)2 * 0.5 * 0.5 / (0.05)2

n = 384.16 ≈ 385

The sample size required to achieve a 95% confidence interval with a 5% margin of error is 385.

b) Since a recent survey suggests that about 78% of adults have heard of the brand, we can use this value for p instead of 0.5. The formula for the sample size becomes:

n= z2 * p * q / E2



Where:
z = the z-score
p = the sample proportion
q = 1-p
E = the margin of error
n = the sample size

z is the z-score associated with the desired confidence level. For a 95% confidence level, the z-score is 1.96. Hence:

n = (1.96)2 * 0.78 * 0.22 / (0.05)2

n = 371.41 ≈ 372

The sample size required to achieve a 95% confidence interval with a 5% margin of error is 372.

To achieve a representative sample, the survey should be conducted on adults from diverse backgrounds and regions to ensure a range of opinions are captured.

To know more about range visit:

https://brainly.com/question/29204101

#SPJ11

The results from a statistics class’ first test are as follows: The average grade obtained on the test by its 45 students is 85, with a standard deviation of 15 points. Answer the following based on this information:
Approximately how many people received a failing grade (less than 65)?
The results from a statistics class’ first test are as follows: The average grade obtained on the test by its 45 students is 85, with a standard deviation of 15 points. Answer the following based on this information:
What percentage of people received a grade between 70 and 91?
The results from a statistics class’ first test are as follows: The average grade obtained on the test by its 45 students is 85, with a standard deviation of 15 points. Answer the following based on this information:
What percentage of individuals received a score whose z-score was -.70 or less?
The results from a statistics class’ first test are as follows: The average grade obtained on the test by its 45 students is 85, with a standard deviation of 15 points. Answer the following based on this information:
What grade is required in order to be in the top 30 percent?
The results from a statistics class’ first test are as follows: The average grade obtained on the test by its 45 students is 85, with a standard deviation of 15 points. Answer the following based on this information:
What grade is required in order to be in the top 22 percent?

Answers

We are given the average grade as 85 and the standard deviation as 15 points. Using these, we need to find out various percentages of students in the class based on the given conditions, which are explained below:The mean of the class is 85, and standard deviation is 15.

The score which is less than 65 will be calculated using the z-score formula as:

z = (x - μ) / σ

Where x = 65, μ = 85, and σ = 15Substituting the values, we have

z = (65 - 85) / 15z = -1.33

The probability of the score being less than 65 is given by the probability of getting a z-score less than -1.33. Using the z-table, we can find the area as 0.0912, which can be multiplied by the total number of students to get the number of students that got a failing grade.Approximately 4 students received a failing grade (less than 65). We are given the results of the first test in a statistics class. We have to find out various percentage values based on the data given in the question. The mean value is 85, and the standard deviation is 15 points. By using the formula for z-score, we can find out the percentage of students who got grades less than or greater than a certain value. For instance, to find out the percentage of students who scored between 70 and 91, we first need to calculate the z-score for these values.The z-score for a value of 70 is:

z = (x - μ) / σ= (70 - 85) / 15= -1

The z-score for a value of 91 is:

z = (x - μ) / σ= (91 - 85) / 15= 0.4

We then find the probability of getting a value between these two z-scores. We use the standard normal distribution table to find this value. We know that the probability of getting a z-score between -1 and 0.4 is 0.4222. This value multiplied by the total number of students will give us the number of students who scored between 70 and 91. We can use a similar method to find out the number of students that received a score whose z-score was -.70 or less.To find the grade required to be in the top 30%, we first need to find out the z-score that corresponds to this percentile. We know that the area to the left of a z-score of 0.52 is 0.6997. Therefore, the area to the right of this z-score is 0.3003, which corresponds to the top 30% of the class. We then use the formula for z-score to find the corresponding grade value as:

z = (x - μ) / σ0.52 = (x - 85) / 15x = (0.52 * 15) + 85x = 93.8

Therefore, the grade required to be in the top 30% is 93.8.To find the grade required to be in the top 22%, we first need to find out the z-score that corresponds to this percentile. We know that the area to the left of a z-score of 0.81 is 0.7902. Therefore, the area to the right of this z-score is 0.2098, which corresponds to the top 22% of the class. We then use the formula for z-score to find the corresponding grade value as:

z = (x - μ) / σ0.81 = (x - 85) / 15x = (0.81 * 15) + 85x = 96.15

Therefore, the grade required to be in the top 22% is 96.15.

To summarize, we used the given mean and standard deviation values to find out various percentages of students based on different conditions. We calculated the number of students that received a failing grade, the number of students that received a grade between 70 and 91, the number of students that received a score whose z-score was -.70 or less, the grade required to be in the top 30%, and the grade required to be in the top 22%.

To learn more about z-table visit:

brainly.com/question/26836799

#SPJ11

This is for a final pleasd help​

Answers

A. Factorising 3x¹⁰  -  48x² using the greatest common factor is 3x²(x⁸ - 16).

B. Factorising completely is 3x²( (x²- 2)(x² + 2)(x² + 2 - 2x)(x² + 2 + 2x))

How to factorise an expression?

To factorize an expression, the highest common factors of the terms of the given expression are determined and then we group the terms accordingly.

Therefore, let's factorise using the greatest common factor of the expression as follows;

3x¹⁰  -  48x²

Hence, the greatest common factor is 3x²

Therefore,

3x¹⁰  -  48x² = 3x²(x⁸ - 16)

B.

Therefore, let's factor the expression completely,

3x¹⁰  -  48x² = 3x²(x⁸ - 16)

Then,

(x⁸ - 16) = (x⁴ + 4)(x⁴ - 4) = (x²- 2)(x² + 2)(x² + 2 - 2x)(x² + 2 + 2x)

Hence,

3x¹⁰  -  48x² = 3x²( (x²- 2)(x² + 2)(x² + 2 - 2x)(x² + 2 + 2x))

learn more on expression here:https://brainly.com/question/22048684

#SPJ1

Write the algebraic expression for the following: a. t in S=P(1+rt) 1) t= S/P²r 2) t= S−P/Pr 3) t= SPr/P
b. d in N=L(1−d) 1) d= N−L/L 2) d=− N−L/L 3) d= −N−L/L

Answers

Algebraic expression are:-

a. t = S/(P^2r) - 1/r

b. d = (N - L)/L

a. To find the algebraic expression for "t" in the equation S = P(1 + rt), we can solve for "t" by manipulating the equation.

1) t = S/(P^2r)

To isolate "t", divide both sides of the equation by P(1 + rt):

S = P(1 + rt)

S/P = 1 + rt

S/P - 1 = rt

t = (S/P - 1)/r

t = S/(P^2r) - 1/r

2) t = (S - P)/(Pr)

In this case, we can start by dividing both sides of the equation by P:

S/P = 1 + rt

(S - P)/P = rt

t = (S - P)/(Pr)

3) t = SPr/P

Similarly, by dividing both sides of the equation by Pr:

S = P(1 + rt)

S/Pr = 1 + rt

SPr/P = rt

t = SPr/P

b. To find the algebraic expression for "d" in the equation N = L(1 - d), we can follow a similar process.

1) d = (N - L)/L

To isolate "d", divide both sides of the equation by L:

N = L(1 - d)

N/L = 1 - d

d = (N - L)/L

2) d = - (N + L)/L

In this case, we can start by dividing both sides of the equation by -L:

N = L(1 - d)

-N/L = 1 - d

d = - (N + L)/L

3) d = -N/(N + L)

Similarly, by dividing both sides of the equation by (N + L):

N = L(1 - d)

-N/(N + L) = 1 - d

d = -N/(N + L)

These algebraic expressions provide different forms for the variables "t" and "d" in terms of the given equations, allowing for different ways to represent the relationship between the variables in each scenario.

Learn more about Algebraic expression here :-

https://brainly.com/question/28884894

#SPJ11

In 1960 the world record for the men's mile was 3.91 minutes. In 1980, the record time was 3.81 minutes. Write a linear model that represents the world record for the men's mile as a function of the number of years since 1960.Use the model to estimate the record time in 2000 and predict the record time in 2020.

Answers

The record time in 2000 is, 3.71 minutes

We have,

In 1960 the world record for the men's mile was 3.91 minutes. In 1980, the record time was 3.81 minutes.

Here, A line passes through the points (0,3.91) and (20,3.81).

Hence, the slope of the line is,

m = (3.81 - 3.91) / (20 - 0)

m = - 0.1/20
m = - 0.005

Thus, the equation of a line is,

y - 3.91 = - 0.005 (x - 0)

y - 3.91 = - 0.005x

y = - 0.005x + 3.91

So, the record time in 2000 is,

Put x = 40;

y = - 0.005 × 40 + 3.91

y = - 0.2 + 3.91

y = 3.71 minutes

To learn more about the equation of line visit:

https://brainly.com/question/18831322

#SPJ4

Simplify the following (minimum shown in parenthesis.): xyz + xyz' + x'yz' + x'y'z (3 terms, 7 literals)

Answers

The concept of simplifying a Boolean expression involves reducing the expression to its most concise form by applying logical rules and simplification techniques. This helps in reducing complexity, improving readability, and optimizing logic circuits by eliminating redundant terms and literals.

The simplified expression consists of two terms with a total of 5 literals.

To simplify the expression:

xyz + xyz' + x'yz' + x'y'z

We can apply Boolean algebra rules to simplify the terms:

Combine terms with common literals:

xyz + xyz' = xy(z + z') = xy

Combine terms with common literals:

x'yz' + x'y'z = x'z(y + y') = x'z

Now we have simplified the expression to:

xy + x'z

To know more about Boolean algebra refer to-

https://brainly.com/question/31647098

#SPJ11

A group of adult males has foot lengths with a mean of 27.23 cm and a standard deviation of 1.48 cm. Use the range rule of thumb for identifying significant values to identify the limits separating values that are significantly low or significantly high. Is the adult male foot length of 23.7 cm significantly low or significantly high? Explain. Significantly low values are cm or lower. (Type an integer or a decimal. Do not round.) Significantly high values are cm or higher. (Type an integer or a decimal. Do not round.) Select the correct choice below and fill in the answer box(es) to complete your choice. A. The adult male foot length of 23.7 cm is significantly low because it is less than cm. (Type an integer or a decimal. Do not round.) B. The adult male foot length of 23.7 cm is not significant because it is between cm and cm. (Type integers or decimals. Do not round.) C. The adult male foot length of 23.7 cm is significantly high because it is greater than cm. (Type an integer or a decimal. Do not round.)

Answers

The range rule of thumb is used to estimate data spread by determining upper and lower limits based on the interquartile range (IQR). It helps identify significantly low and high values in foot length for adult males. By calculating the z-score and subtracting the product of the standard deviation and range rule of thumb from the mean, it can be determined if a foot length is significantly low. In this case, a foot length of 23.7 cm is deemed significantly low, supporting option A.

The range rule of thumb is an estimation technique used to evaluate the spread or variability of a data set by determining the upper and lower limits based on the interquartile range (IQR) of the data set. It is calculated using the formula: IQR = Q3 - Q1.

Using the range rule of thumb, we can find the limits for significantly low values and significantly high values for the foot length of adult males.

The limits for significantly low values are cm or lower, while the limits for significantly high values are cm or higher.

To determine if a foot length of 23.7 cm is significantly low or high, we can use the mean and standard deviation to calculate the z-score.

The z-score is calculated as follows:

z = (x - µ) / σ = (23.7 - 27.23) / 1.48 = -2.381

To find the lower limit for significantly low values, we subtract the product of the standard deviation and the range rule of thumb from the mean:

27.23 - (2.5 × 1.48) = 23.7

The adult male foot length of 23.7 cm is considered significantly low because it is less than 23.7 cm. Therefore, option A is correct.

To know more about range rule of thumb Visit:

https://brainly.com/question/33321388

#SPJ11

The following represent statistics of weekly salaries at Acme Corporation. Mean =$585 Median =$581 Mode =$575 Standard deviation =$28 First Quartile =$552 Third Quartile =$60586 th Percentile =$612P 64

=$592 a) What is the most common salary? b) What salary did half the employee's salaries surpass? c) About what percent of employee's salaries is below $612? d) What percent of the employee's salaries are above $552? e) What salary is 2 standard deviations below the mean? f) About what percent of employee's salaries is above $592 ? g) What salary is 1.5 standard deviations above the mean? h) Assume the weekly salaries have a distribution that is bell-shaped, about what percent of the salaries would be between $529 and $641 ?

Answers

h) Assuming a bell-shaped distribution, approximately 68% of the salaries would fall within one standard deviation of the mean. Therefore, we can estimate that about 68% / 2 = 34% of the salaries would be between $529 and $641.

a) The most common salary, or the mode, is $575.

b) The median salary is $581. This means that half of the employee's salaries surpass $581.

c) Approximately 64% of employee's salaries are below $612. This is indicated by the 64th percentile value.

d) The first quartile is $552, which represents the 25th percentile. Therefore, approximately 25% of the employee's salaries are above $552.

e) Two standard deviations below the mean would be calculated as follows:

  2 * $28 (standard deviation) = $56

  Therefore, the salary that is 2 standard deviations below the mean is $585 - $56 = $529.

f) About 50% of the salaries are above the median, so approximately 50% of employee's salaries are above $592.

g) 1.5 standard deviations above the mean would be calculated as follows:

  1.5 * $28 (standard deviation) = $42

  Therefore, the salary that is 1.5 standard deviations above the mean is $585 + $42 = $627.

To know more about deviations visit:

brainly.com/question/13498201

#SPJ11

Use the graph of F to find the given limit. When necessary, state that the limit does not exist.
lim F(x)
X-4
Select the correct choice below and, if necessary, fill in the answer box to complete your choice.
OA. lim F(x)= x-4 (Type an integer or a simplified fraction.)
OB. The limit does not exist.

Answers

The limit of the function in this problem is given as follows:

[tex]\lim_{x \rightarrow 4} F(x) = 5[/tex]

How to obtain the limit of the function?

The graph of the function is given by the image presented at the end of the answer.

The function approaches x = 4 both from left and from right at y = 5, hence the limit of the function is given as follows:

[tex]\lim_{x \rightarrow 4} F(x) = 5[/tex]

The limit would not exist if the lateral limits were different.

More can be learned about limits at brainly.com/question/23935467

#SPJ1

If -12 is an element in the domain of f(x)=6(8-x), what is its corresponding element in the range?

Answers

For the function f(x) = 6(8 - x), if -12 is an element in the domain, its corresponding element in the range can be found by substituting -12 into the function. The corresponding element in the range is 120.

Given the function f(x) = 6(8 - x), we are told that -12 is an element in the domain of the function. To find its corresponding element in the range, we substitute -12 into the function:

f(-12) = 6(8 - (-12))

= 6(8 + 12)

= 6(20)

= 120

Therefore, when -12 is an element in the domain of f(x) = 6(8 - x), its corresponding element in the range is 120. This means that when x = -12, the output of the function is 120.\

To know more about corresponding element  refer here:

https://brainly.com/question/29536704

#SPJ11

Select all the statements that are true about squares.

A. Diagonals are congruent to sides.

B. Diagonals are perpendicular.

C. Consecutive angles are supplementary.

D. Diagonals bisect angles.

E. Opposite sides are parallel.

Answers

The true statements about squares are:

B. Diagonals are perpendicular.

C. Consecutive angles are supplementary.

E. Opposite sides are parallel.

A. Diagonals are congruent to sides: This statement is not true for all squares. In a square, the diagonals are not necessarily congruent to the sides. They are equal in length, but they are not congruent unless the square is also a rhombus.

B. Diagonals are perpendicular: This statement is true for all squares. The diagonals of a square are always perpendicular to each other, forming right angles at their point of intersection.

C. Consecutive angles are supplementary: This statement is true for all squares. In a square, the consecutive angles (adjacent angles) are always supplementary, meaning that their measures add up to 180 degrees. Each angle in a square measures 90 degrees, and the sum of any two consecutive angles is 180 degrees.

D. Diagonals bisect angles: This statement is not true for all squares. The diagonals of a square do not necessarily bisect the angles of the square. They do bisect each other, dividing the square into four congruent right triangles, but they do not necessarily bisect the angles.

E. Opposite sides are parallel: This statement is true for all squares. In a square, opposite sides are always parallel. All sides of a square are equal in length, and opposite sides are parallel to each other.

Learn more about   statement  from

https://brainly.com/question/27839142

#SPJ11

Prove That For All Sets A And B, If A×B=B×A, Then A=B Or A=∅ Or B=∅

Answers

We have shown that if A and B are non-empty sets, and A≠B, A≠∅, and B≠∅, then there exist elements in A×B that are not in B×A.

To prove that for all sets A and B, if A×B=B×A, then A=B or A=∅ or B=∅, we will use proof by contradiction. That is, we will assume that A and B are non-empty sets, and that A≠B, A≠∅, and B≠∅, and show that this leads to a contradiction with the assumption that A×B=B×A.

Assume that A and B are non-empty sets, and that A≠B, A≠∅, and B≠∅. Then there exists an element a in A that is not in B, or an element b in B that is not in A.

Without loss of generality, assume that there exists an element a in A that is not in B. Then for any element b in B, the ordered pair (a,b) is in A×B, but not in B×A, since (a,b) is not of the form (x,y) where x is in B and y is in A.

Therefore, we have shown that if A and B are non-empty sets, and A≠B, A≠∅, and B≠∅, then there exist elements in A×B that are not in B×A. This contradicts the assumption that A×B=B×A, and therefore we must have either A=B, A=∅, or B=∅.

learn more about non-empty sets here

https://brainly.com/question/1581607

#SPJ11

Find the annual percentage rate compounded continuously to the nearest tenth of a percent for which $20 would grow to $40 for each of the following time periods. a. 5 years b. 10 years c. 30 years d. 50 years a. The sum of $20 would grow to $40 in 5 years, it the antual rate is approximatedy (Do not round until the final anower. Then round to one decimal place as needed.)

Answers

To determine the annual percentage rate (APR) compounded continuously for which $20 would grow to $40 over different time periods, we can use the formula for continuous compound interest. For a 5-year period, the approximate APR can be calculated as [value] percent (rounded to one decimal place).

The formula for continuous compound interest is A = P * e^(rt), where A is the final amount, P is the principal (initial amount), e is the base of the natural logarithm, r is the annual interest rate (as a decimal), and t is the time period in years.

In the given scenario, we have A = $40 and P = $20 for a 5-year period. By substituting these values into the continuous compound interest formula, we obtain $40 = $20 * e^(5r). To solve for the annual interest rate (r), we isolate it by dividing both sides of the equation by $20 and then taking the natural logarithm of both sides. This yields ln(2) = 5r, where ln denotes the natural logarithm.

Next, we divide both sides by 5 to isolate r, resulting in ln(2)/5 = r. Using a calculator to evaluate this expression, we find the value of r, which represents the annual interest rate.

Finally, to express the APR as a percentage, we multiply r by 100. The calculated value rounded to one decimal place will give us the approximate APR compounded continuously for the 5-year period.

To know more about annual percentage rate refer here:

https://brainly.com/question/28347040

#SPJ11

a movies theater is filled with 500 people. After the movie ends people start leave t a rte 50 each minute

Answers

The total number of people in the movie theater is 500. After the movie ends, the people start to leave at a rate of 50 each minute. To determine the time it takes for all of the people to leave the theater, we need to divide the total number of people by the rate at which they are leaving.

This is because the rate of people leaving is the number of people leaving in a given time period, so the total time it takes for everyone to leave can be determined by dividing the total number of people by the rate. Therefore, it will take 10 minutes for everyone to leave the movie theater. This is because: Total people in theater

= 500Rate of people leaving

= 50 people per minute Time to exit for all people

= (Total people in theater / Rate of people leaving)

= (500 / 50)

= 10Therefore, it will take 10 minutes for everyone to leave the movie theater.

To know more about variables visit:

https://brainly.com/question/15078630

#SPJ11

Can someone please look at my script and explain why the data is not being read and entered into my pretty table? Any help is appreciated. Script is below. I am getting an empty pretty table as my output.
# Python Standard Library
import os
from prettytable import PrettyTable
myTable = PrettyTable(["Path", "File Size", "Ext", "Format", "Width", "Height", "Type"])
dirPath = input("Provide Directory to Scan:") i
f os.path.isdir(dirPath):
fileList = os.listdir(dirPath)
for eachFile in fileList:
try:
localPath = os.path.join(dirPath, eachFile)
absPath = os.path.abspath(localPath)
ext = os.path.splitext(absPath)[1]
filesizeValue = os.path.getsize(absPath)
fileSize = '{:,}'.format(filesizeValue)
except:
continue
# 3rd Party Modules from PIL
import Image imageFile = input("Image to Process: ")
try:
with Image.open(absPath) as im: #
if success, get the details imStatus = 'YES'
imFormat = im.format
imType = im.mode
imWidth = im.size[0]
imHeight = im.size[1]
#print("Image Format: ", im.format)
#print("Image Type: ", im.mode)
#print("Image Width: ", im.width)
#print("Image Height: ", im.height)
except Exception as err:
print("Exception: ", str(err))
myTable.add_row([localPath, fileSize, ext, imFormat, imWidth, imHeight, imType])
print(myTable.get_string())

Answers

The data is not being read file and entered into the pretty table because there is a name error, `imFormat`, `imType`, `imWidth`, and `imHeight` are not declared in all cases before their usage. Here is the modified version of the script with corrections:```
# Python Standard Library
import os
from prettytable import PrettyTable
from PIL import Image

myTable = PrettyTable(["Path", "File Size", "Ext", "Format", "Width", "Height", "Type"])
dirPath = input("Provide Directory to Scan:")
if os.path.isdir(dirPath):
   fileList = os.listdir(dirPath)
   for eachFile in fileList:
       try:
           localPath = os.path.join(dirPath, eachFile)
           absPath = os.path.abspath(localPath)
           ext = os.path.splitext(absPath)[1]
           filesizeValue = os.path.getsize(absPath)
           fileSize = '{:,}'.format(filesizeValue)
       except:
           continue

       # 3rd Party Modules from PIL
       imageFile = input("Image to Process: ")
       try:
           with Image.open(absPath) as im:
               # If successful, get the details
               imStatus = 'YES'
               imFormat = im.format
               imType = im.mode
               imWidth = im.size[0]
               imHeight = im.size[1]
       except Exception as err:
           print("Exception: ", str(err))
           continue
       myTable.add_row([localPath, fileSize, ext, imFormat, imWidth, imHeight, imType])

   print(myTable)
```The above script now reads all the images in a directory and outputs details like format, width, and height in a pretty table.

To know more about read a file refer here :

https://brainly.com/question/31670046#

#SPJ11

Suppose That F(X)=4x+7,G(X)=X, And H(X)=9x−5 Find (F∘G∘H)(X). (F∘G∘H)(X)=

Answers

To find (F∘G∘H)(X), we need to evaluate the composition of the three functions: F(G(H(X))).

First, let's evaluate H(X) by substituting X into the expression: H(X) = 9X - 5.

Next, we evaluate G(H(X)) by substituting H(X) into the expression for G: G(H(X)) = G(9X - 5) = 9X - 5.

Finally, we evaluate F(G(H(X))) by substituting G(H(X)) into the expression for F: F(G(H(X))) = F(9X - 5) = 4(9X - 5) + 7 = 36X - 13.

Therefore, (F∘G∘H)(X) = 36X - 13.

Learn more about functions here: brainly.com/question/30660139

#SPJ11

(CLO3) (a) There are 3 Bangladeshis, 4 Indians, and 5 Pakistanis available to form a committee consisting of a president, a vice-president, and a secretary. In how many ways can a committee be formed given that the three members must be from three different countries?

Answers

Therefore, there are 60 ways to form the committee with one person from each country.

To form the committee with a president, a vice-president, and a secretary, we need to select one person from each country.

Number of ways to select the president from Bangladeshis = 3

Number of ways to select the vice-president from Indians = 4

Number of ways to select the secretary from Pakistanis = 5

Since the members must be from three different countries, the total number of ways to form the committee is the product of the above three selections:

Total number of ways = 3 * 4 * 5 = 60

Learn more about committee  here

https://brainly.com/question/31624606

#SPJ11

Solve \frac{x^{2}-6 x+9}{2 x^{3}+19 x^{2}+40 x-25}<0 and show your answer on a number line.

Answers

The solution to the inequality, $\frac{x^{2}-6 x+9}{2 x^{3}+19 x^{2}+40 x-25}<0$, is $x\in(-5,\frac{1}{2})$. On the number line, we can mark -5 and 1/2 with open circles. We can shade the interval between -5 and 1/2, excluding the endpoints.

The given inequality is: $\frac{x^{2}-6 x+9}{2 x^{3}+19 x^{2}+40 x-25}<0$To solve the inequality and show the answer on a number line, we can follow the given steps:

Step 1: Find the critical values that make the denominator zero. In other words, solve $2 x^{3}+19 x^{2}+40 x-25=0$ for x.  Factorizing the expression:$(x+5)(2x-1)(x+5)=0$x = -5, 1/2 are the critical values.

Step 2: Divide the number line into four parts, with critical values as endpoints. -5 and 1/2 divide the line into 3 intervals: $(-∞,-5)$, $(-5, 1/2)$ and $(1/2,∞)$.

Step 3: Choose any value within each of the intervals and test it in the inequality. If the result is true, then all the values within that interval satisfy the inequality. If the result is false, then none of the values within that interval satisfy the inequality.  We can use the sign table to find the sign of the expression $\frac{x^{2}-6 x+9}{2 x^{3}+19 x^{2}+40 x-25}$. $$\begin{array}{|c|c|c|c|} \hline \textbf{Intervals} & x<-5 & -5\frac{1}{2} \\ \hline x^{2}-6x+9 & + & + & +\\ \hline 2x^{3}+19x^{2}+40x-25 & - & + & + \\ \hline \frac{x^{2}-6 x+9}{2 x^{3}+19 x^{2}+40 x-25} & - & + & - \\ \hline \end{array}$$

Step 4: Show the sign of the expression within each interval on the number line as follows: From the sign table, the inequality is satisfied when: $\frac{x^{2}-6 x+9}{2 x^{3}+19 x^{2}+40 x-25}<0$ for $x\in(-5,\frac{1}{2})$. Therefore, the solution to the inequality, $\frac{x^{2}-6 x+9}{2 x^{3}+19 x^{2}+40 x-25}<0$, is $x\in(-5,\frac{1}{2})$.

Therefore, the answer is given as follows: On the number line, we can mark -5 and 1/2 with open circles. We can shade the interval between -5 and 1/2, excluding the endpoints. The solution set is represented by this shaded interval.

To know more about number line: https://brainly.com/question/28468614

#SPJ11

Write a slope -intercept equation for a line passing through the point (6,-6) that is parallel to the line x=-7

Answers

The equation for the line passing through the point (6,-6) that is parallel to the line x=-7 is x=6.

To find the slope-intercept equation for a line passing through the point (6,-6) that is parallel to the line x=-7, we first need to find the slope of the given line x=-7. The given equation x=-7 represents a vertical line passing through the point (-7, y) for all values of y.

Therefore, the slope of the given line is undefined or infinite. This means any line that is parallel to this line will also have an undefined slope. So, the equation for the parallel line will be x = a, where a is a constant. To find the value of a, we will use the point (6, -6) that the parallel line passes through.

Therefore, the equation of the parallel line is x = 6. The slope-intercept form of a line is y = mx + b, where m is the slope of the line and b is the y-intercept. Since the slope of the parallel line is undefined, there is no slope-intercept equation for this line. Thus, x = 6 is the final answer.To summarize, the equation for the line passing through the point (6,-6) that is parallel to the line x=-7 is x=6. The reason is that the given equation represents a vertical line passing through the point (-7, y) for all values of y.

This means that any line parallel to this line will also have an undefined slope or an infinite slope. Therefore, the equation for the parallel line will be x = a, where a is a constant. To find the value of a, we used the point (6, -6) that the parallel line passes through. We concluded that the equation of the parallel line is x = 6. Since the slope of the parallel line is undefined, there is no slope-intercept equation for this line. So, the final answer is x = 6.

To know more about parallel line visit :

https://brainly.com/question/29762825

#SPJ11

What is the area of this rectangle? Rectangle with width 5. 1 cm and height 11. 2 cm. Responses 16. 3 cm2 16. 3 cm, 2 32. 6 cm2 32. 6 cm, 2 57. 12 cm2 57. 12 cm, 2 571. 2 cm2

Answers

The area of the rectangle is 57.12 cm^2.

The area of a rectangle is the product of its length or height and width. The formula for calculating the area of a rectangle is:

Area = Width x Height

In this problem, we are given the width of the rectangle as 5.1 cm and the height as 11.2 cm. To find the area, we substitute these values into the formula to get:

Area = 5.1 cm x 11.2 cm

Area = 57.12 cm^2

Therefore, the area of the rectangle is 57.12 square centimeters (cm^2).

Learn more about area  from

https://brainly.com/question/25292087

#SPJ11

Find the length of the following two-dimensional curve. r(t)=⟨5t^2 −2,12t^2
+3⟩, for 0≤t≤1 The arc length is L=

Answers

The arc length of the two-dimensional curve, calculated using the given formula and values, is found to be 13 units.

The arc length formula for the two-dimensional curve is given by; [tex]L = \int ab\sqrt(dx/dt)^2+(dy/dt)^{2dt}[/tex], Where; dx/dt = 10t and dy/dt = 24t

The length of the two-dimensional curve can be found using the arc length formula as shown below:

[tex]L = \int ab\sqrt(dx/dt)^2+(dy/dt)^{2dt}[/tex]

[tex]L = \int_0^1\sqrt(10t)^2+(24t)^{2dt}[/tex]

[tex]L = \int_0^1\sqrt(100t^2+576t^2)dt[/tex]

[tex]L = \int_0^1 \sqrt(676t^2)dt = L = \int_0^126t dt[/tex]

[tex]L = 13t^2[_0^1]L = 13(1)^2 - 13(0)^2 = L = 13[/tex]

Therefore, the length of the two-dimensional curve is 13.

Arc length is the measure of the distance along the curved line of an arc. It represents the portion of the circumference of a circle or the boundary of any curved shape. It is calculated by multiplying the angle subtended by the arc (in radians) with the radius of the circle.

Arc length is an important concept in geometry and is used in various fields such as physics, engineering, and architecture for accurate measurements and calculations.

For more questions on arc length

https://brainly.com/question/28108430

#SPJ8

If
2oz is 190 calories, how many calories is 2.5 oz?
Please explain answer.

Answers

2.5 oz of the given food contains 237.5 calories.

To solve the given problem, first we need to know the unitary method of solving the problem involving ratio and proportion.

Unitary method is the method of solving the problems in which we find the value of one unit first and then multiply it to find the required value. It is used to find the value of a unit, when the value of another unit is given.

So, to solve the given problem, we need to first find the value of 1 oz.

Let x be the number of calories in 1 oz of the given food.

Then we can say that,2 oz of the food has = 2x calories. (According to given data, 2 oz is 190 calories)

To find the calories in 2.5 oz of the food, we can use the unitary method;

Number of calories in 1 oz = x

Number of calories in 2 oz = 2x

Number of calories in 2.5 oz = 2.5x calories

We can use the proportionality concept of unitary method;

So, 2 oz of the food has = 2x calories.

1 oz of the food has = x calories.

Thus, 2 oz of the food has = 2 times the calories in 1 oz of the food.

Hence, the number of calories in 1 oz of the food is 190/2 = 95 calories.

So, Number of calories in 2.5 oz of the food = 2.5 times the calories in 1 oz of the food

= 2.5 × 95 calories

= 237.5 calories.

Therefore, 2.5 oz of the given food contains 237.5 calories.

Learn more about calories: https://brainly.com/question/28589779

#SPJ11

Other Questions
Raider Investments completed the following investment transactions during 2024: 1) (Click the icon to view the transactions.) Requirements 1. Journalize the entries for 2024 . Explanations are not required. 2. What account(s) and amount(s), if any, would be reported on Raider's income statement for the year ended December 31, 2024? Requirement 1. Joumalize the entries for 2024. Explanations are not required. (Record debits first, then credits. Exclude explanations from journal entries. If no entry is required, select "No entry required" on the first line of the Accounts column and leave the remaining cells blank.) Jan. 14: Purchased 700 shares of Robotic stock, paying $55 per share. The investment represents 2% ownership in Robotic's voting stock. Raider does not have significant influence over Robotic. Raider intends to hold the investment for the indefinite future. Aug. 22: Received a cash dividend of $0.26 per share on the Robotic stock. Dec. 31: Adjusted the Robotic inventment to its current market value of $59. More info Jan. 14 Purchased 700 shares of Robotic stock, paying $55 per share. The investment represents 2\% ownership in Robotic's voting stock. Raider does not have significant influence over Robotic. Raider intends to hold the investment for the indefinite future. Aug. 22 Received a cash dividend of $0.26 per share on the Robotic stock. Dec.31 Adjusted the Robotic investment to its current market value of $59. Requirement 2. What account(s) and amount(s), if any, would be reported on Raider's income statement for the year ended December 31,2024? (Use parentheses or a minus sign for a loss.) What is the equation of the line that passes through the points (-2, -4) and (-3, -5)? Write your answer in slope -intercept form. {ClO}_{3}{ }^{-}+{SnO}_{2}^{2} {SnO}_{2}^{2}+{ClO}_{4}^{-} In the above redox reaction, use oxidation numbers to identify the element oxidized, the elemen The time (in minutes) until the next bus departs a major bus depot follows a distribution with f(x)=1/20, where x goes from 25 to 45 minutes.P(25 < x < 55) = _________.10.90.80.20.10 your semester project is to design and implement the database that could drive a coronavirus tracking application. ACTIVE LEARNING TEMPLATE: Therapeutic Procedure STUDENT NAME PROCEDURE NAME REVIEW MODULE CHAPTER Description of Procedure Indications CONSIDERATIONS Nursing Interventions (pre, intra, post) Outcomes/Evaluation Client Education Potential Complications Nursing Interventions ATMF I FAPS NA TERATES 1 At the campus coffee cart, a medium coffee costs $3.35. Mary Anne brings $4.00 with her when she buys a cup of coffee and leaves the change as a tip. What percent tip does she leave? The ground plane of the 3D environment is displayed in the 3D grid. As implied by the name, the ground plane is a plane that is affixed to the ground of the scene, where Y is equal to 0. The boundary between up and down, or between positive and negative Y values, is represented by the ground plane. It is centered on (0, 0, 0). Current ratio Days' sales in inventory Debt ratio Dividends per share Earnings per share Gross profit percentage Inventory turnover Return on assets Return on common stockholders' equity Return on sales Requirement 3. Evaluate the ability to sell merchandise inventary. Begin by selecting the appropriate measurements that should be used to measure the ability to sell inventory and the profitablity of each sales dolar above the cost of goods scid. fourthBits - return an int with every 4th bit set to 1 Example: fourthBits ()=2004318072 Hint: Each hex digit will have the MSB=1 and other bits as 0 Legal ops: !&+> Max ops: 8 Rating: 2 / int fourthBits(void) \{ return 2; \} the most successful retailers anticipate change and develop new ways to interact with customers. true or false? inbound marketing is a strategy to direct referrals into the organization. Use the following to answer questions 7-9:Use this scenario to answer questions 32-36.Scenario: Open Economy S = IIn an open economy suppose that GDP is $12 trillion. Consumption is $8 trillion and government spending is $2 trillion. Taxes are $0.5 trillion. Exports are $1 trillion and imports are $3 trillion.(Scenario: Open Economy S = I) How much is private saving?$3.5 trillion s 7.421 g of carbon, 0.779 g of hydrogen, 4.329 g of nitrogen, and 2.472 g of oxygen. the empirical formula of caffeine is richman investments is concerned about the security of its customer data. management has determined that the three primary risks the company faces in protecting the data are as follows: When Hurton ak Koula for a firt-peron account of hi life in Africa, Koula repond with "where i de houe where de moue i de leader? In de Afficka oil I cain tellee you bout de on before I tellee you bout de father. " Paraphrae what Koula mean by thi in your own word. Evaluate hi deciion to tructure hi tory in a chronological manner. How doe thi repone demontrate the value that are inherent in Koula native culture? Support your analyi with evidence from the text Bottom-up changes sometimes originate from managers. Indicate whether the statement is true or false Following is the chemical structure of a common medication.Evaluate the structure and determine the number ofsp2 and sp3 hybridizedcarbons present.Ibuprofensp2:sp3: Give an efficient algorithm that partitions the numbers into n pairs, with the property that the partition minimizes the maximum sum of a pair. For example, say we are given the numbers (2,3,5,9). The possible partitions are ((2,3),(5,9)), ((2,5),(3,9)), and ((2,9),(3,5)). The pair sums for these partitions are (5,14),(7,12), and (11,8). Thus the third partition has 11 as its maximum sum, which is the minimum over the three partitions Need discussion post regarding amazon capital investments have associated cash flows explaining it . Name two capital investments of amazon and how they differ with estimated cash flow with a reference